A Galapagos tortoise can 1/5 mile in an hour. How many hours would it take the tortoise to walk 1/6?

Answers

Answer 1

Answer:

5/6 hours

Step-by-step explanation:

Create a proportion where x is the number of hours it will take to walk 1/6 of a mile:

[tex]\frac{1/5}{1}[/tex] = [tex]\frac{1/6}{x}[/tex]

Cross multiply and solve for x:

1/5x = 1/6

x = 5/6

So, it will take the tortoise 5/6 hours to walk 1/6 of a mile


Related Questions

3. If a + b = C, which of the following statements is true?

Answers

Answer:

first one

Step-by-step explanation:

a + b = c

subtract a from both sides

b = c - a

or

c - a = b

[tex]\longrightarrow{\green{ a. \: c - a = b }}[/tex] 

[tex]\large\mathfrak{{\pmb{\underline{\red{Step-by-step\:explanation}}{\red{:}}}}}[/tex]

[tex]a + b = c[/tex]

[tex]⇢ b = c - a[/tex]

[tex]⇢ c - a = b[/tex]

[tex]\large\mathfrak{{\pmb{\underline{\orange{Mystique35 }}{\orange{❦}}}}}[/tex]

The price of an item has been reduced by $1.84. The original price was $46.91. What is the price now?

Answers

Answer:

$45.07

Step-by-step explanation:

46.91-1.84= 45.07

Un microempresario realiza una inversión donde le pagan una rentabilidad del 5,36% mensual simple, él desea que esa inversión se triplique ¿Cuánto tiempo deberá esperar para que esto ocurra?

Answers

Answer:

Deberá esperar 37.31 años para que esto ocurra.

Step-by-step explanation:

Este es un problema de interés simple.

La fórmula del interés simple viene dada por:

[tex]E = P*I*t[/tex]

En el que E es la cantidad de interés ganado, P es el principal (la cantidad inicial de dinero), I es la tasa de interés (anual, como decimal) y t es el tiempo.

Después de t años, la cantidad total de dinero es:

[tex]T = E + P[/tex]

5,36% mensual simple

Esto implica que [tex]I = 0.0536[/tex]

Inversión se triplique ¿Cuánto tiempo deberá esperar para que esto ocurra?

Tiempo t para que la cantidad de interés sea lo doble de la cantidad inicial, o sea, [tex]E = 2P[/tex].

[tex]E = P*I*t[/tex]

[tex]2P = P*I*t[/tex]

[tex]It = 2[/tex]

[tex]0.0536t = 2[/tex]

[tex]t = \frac{2}{0.0536}[/tex]

[tex]t = 37.31[/tex]

Deberá esperar 37.31 años para que esto ocurra.

What are the coordinates of the point on the directed line segment from ( − 10 , 2 ) (−10,2) to ( − 10 , − 5 ) (−10,−5) that partitions the segment into a ratio of 6 to 1?

Answers

Answer: [tex](-10,-4)[/tex]

Step-by-step explanation:

Given

The points joining line segment are (-10,2) and (-10,-5)

Using section formula, a point joining segment  [tex](x_1,y_1)\ \text{and}\ (x_2,y_2)[/tex] divides it in k:1  has coordinates

[tex]x=\dfrac{kx_2+1\cdot x_1}{k+1}[/tex], [tex]y=\dfrac{ky_2+y_1}{k+1}[/tex]

Ratio is 6:1

Coordinates are

[tex]\Rightarrow x=\dfrac{6(-10)+(-10)}{6+1}\\\\\Rightarrow x=-10[/tex]

[tex]\Rightarrow y=\dfrac{-5(6)+1(2)}{6+1}\\\\\Rightarrow y=\dfrac{-28}{7}\\\\\Rightarrow y=-4[/tex]

Thus, the coordinates are [tex](-10,-4)[/tex]

the koala bear is 21 inches tall. the elephant is 9 times as tall as the koala. How tall is the elephant?

Answers

9 times 21 is 189
the answer is 189 inches

Answer:

189 inches tall

Step-by-step explanation:

se
4 Shonda is hanging pictures on a
wall in her home. She has pictures
with areas of 16 square inches and
25 square inches. She hangs a total
of 7 pictures which covers 148
square inches of wall space. Let x be
the number of pictures with an area
of 16 square inches and y be the
number of pictures with an area of
25 square inches.

Answers

X+y=7 and 16x+25y=148

Please help
Solve this system of inequality by graphing.

Answers

Answer: Thats what the graph would look like hope this helps :)

Step-by-step explanation:

Answer:

see graph.

[tex]\leq[/tex] and [tex]\geq[/tex] are solid lines

< and > are dotted lines

graph inequalities same as regular equations

< shade below

> shade above

Pls someone help me with this it's urgent!

Answers

Answer : square root of x - 5 ,

the answer the the last option choice.

Answer:

I think it is y=√x -5

Step-by-step explanation:

if clt 5wont go with √x

Given d= 4t+1 can you determine the value of the y-
intercept?

HELP!! asap!

Answers

Yes. The y-intercept is 1 since the equation for a line is equal to y=mx+b, where b is the y-intercept.

Work out the sum of 1/4 1/6 and 1/3

Answers

Answer:

[tex]\frac{3}{4}[/tex]

Step-by-step explanation:

[tex]\frac{1}{4} +\frac{1}{6} +\frac{1}{3}[/tex]

4 = 2 × 2 × 1

6 = 2 × 3 × 1

3 = 3 × 1

GCF = 1

4 × 6 × 3 = 72

[tex]\frac{18}{72} +\frac{12}{72} +\frac{24}{72}[/tex]

[tex]\frac{18+12+24}{72}[/tex]

[tex]\frac{54}{72}[/tex]

54 = 2 × 3 × 3 × 3

72 = 2 × 2 × 2 × 3 × 3

GCF = 2 × 3 × 3

GCF = 18

[tex]\frac{\frac{54}{18} }{\frac{72}{18} }=\frac{3}{4}[/tex]

[tex]\huge\textsf{Hey there!}[/tex]

[tex]\large\textsf{Looking for your LCD (Least Common Denominator) of each fraction}\downarrow\\\\\\\mathsf{\dfrac{1}{4}=\dfrac{3}{12}}\\\\\\\mathsf{\dfrac{1}{6}=\dfrac{2}{12}}\\\\\\\mathsf{\dfrac{1}{3}=\dfrac{4}{12}}\\\\\\\\\\\large\textsf{The LCD is \underline{12}}[/tex]

[tex]\mathsf{\dfrac{1}{4}+\dfrac{1}{6}+\dfrac{1}{3}}[/tex]

[tex]\mathsf{= \ \dfrac{1\times3}{4\times3}+\dfrac{1\times2}{6\times2}+\dfrac{1\times4}{3\times4}}[/tex]

[tex]\large\textsf{Solving for the numerators (TOP numbers)}\downarrow\\\\\mathsf{1\times3=\boxed{\bf 3}}\\\\\mathsf{1\times2=\boxed{\bf 2}}\\\\\mathsf{1\times4=\boxed{\bf 4}}[/tex]

[tex]\large\textsf{Solving for the denominators (BOTTOM numbers)}\downarrow\\\\\mathsf{4\times3=\boxed{\bf 12}}\\\\\mathsf{6\times2=\boxed{\bf 12}}\\\\\mathsf{3\times4=\boxed{\bf 12}}[/tex]

[tex]\mathsf{=\ \dfrac{3+2+4}{12}}[/tex]

[tex]\large\textsf{Solving for your numerator (TOP number) }\downarrow\\\\\\\mathsf{3+2+4}\\\\\mathsf{= 5+4}\\\\\mathsf{= \boxed{\large\textsf{\bf 9}}}[/tex]

[tex]\mathsf{=\ \dfrac{9}{12}}[/tex]

[tex]\mathsf{\dfrac{9\div3}{12\div3}}\\\\\\\\\mathsf{9\div3= \boxed{\large\textsf{\bf 3}}}\\\\\mathsf{12\div3=\boxed{\large\textsf{\bf 4}}}[/tex]

[tex]\mathsf{= \dfrac{3}{4}}[/tex]

[tex]\boxed{\boxed{\large\textsf{Answer: }\boxed{\mathsf{\bold{ \dfrac{9}{12}\ or\ \dfrac{3}{4}}\ either\ of\ those\ fractions\ should\ work\ because}}}}\\\boxed{\boxed{\large\textsf{they are equal to each other!}}}\huge\checkmark[/tex]

[tex]\large\textsf{Good luck on your assignment and enjoy your day!}[/tex]

~[tex]\frak{Amphitrite1040:)}[/tex]

If f(x) = 3x - 1 and g(x) = x + 2, find (f- g)(x).
PLZZ HELPPP

Answers

This the answer

If f(x) = 3x - 1 and g(x) = x + 2, find (f- g)(x).

PLZZ HELPPP

2x-3

An apartment has 90 square meters of carpeting. How much is this in square feet? Use the following conversion: 1 square meter is 10.8 square feet.

An explanation on how to do it would also be amazing!

Answers

Answer:

Area = 972 square feet

Step-by-step explanation:

Let  90 square meter be = x square feet

Given :

                      square meter             square feet

                              1                             10.8

                             90                            x

[tex]\frac{1}{90} = \frac{10.8}{x}\\\\1 \times x =10.8 \times 90 \ \ \ \ \ \ \ \ \ \ \ \ \ \ \ \ \ \ [ \ cross \ multiply \ ]\\\\x = 972[/tex]

Therefore , 90 square meter = 972 square feet

Help me pls look at this PHTOo Bc I don’t know how to explain

Answers

Answer:

42.1

Step-by-step explanation:

15.2% = 0.152

277 * 0.152 = 42.104

42.1 megabytes have been downloaded.

Hope this is helpful.

Answer:

42.1

Step-by-step explanation:

thanks for the pts

✌️❤️✌️

A regular hexagon is to be cut out of a circular sheet of metal that has a radius of 6
inches.

Approximately how many square centimeters of sheet metal will be left over as
scraps?
A. 32.9 square centimeters
B. 93.5 square centimeters
C. 113.1 square centimeters
D. 126.2 square centimeters

Answers

The correct answer to your problem would be “d”

Square P and Square Q are similar squares. The ratio of the side length of Square P to the side length of Square Q is 2:7. If the side length of Square P is 3.5 inches, the perimeter of Square Q is?

Answers

Answer:

49 inches

Step-by-step explanation:

since we are talking about squares, and each side in a square has the same length, and all 4 angles are 90 degrees, "similar" did not mean much. every square is similar to every other possible square.

they all are bigger and smaller copies of themselves.

but how much bigger ?

in our case here P is the smaller square and Q the bigger one.

x = side length of P = 3.5"

y = side length of Q

x/y = 2/7

3.5/y = 2/7

7×3.5 = 2y

y = 3.5×3.5 = 12.25"

the perimeter of Q is

4×y = 4×12.25 = 49"


Find the sum of the series
{1875 + 750 + 300+...
+3.072}

Answers

Answer:

Find the indicated sum for each geometric series.

{1875+750+300+⋯+3.072}

Answer: 3122.952

Step-by-step explanation:

{1875+750+300+⋯+3.072}

r=0.4 3.072=1875(0.4)n−1

0.0016384=(0.4)n−1

7=n−1→n=8

Sn=1875(1−(0.4)8)1−0.4=3122.952

I think is like this.

Fill in the blank with <,> or = . 11.01-----11.001

Answers

Answer:  Greater than sign

In other words, 11.01 > 11.001

========================================================

Explanation:

For each given number, move the decimal point 3 spots to the right.

The value 11.01 becomes 11010

The value 11.001 becomes 11001

Since 11010 > 11001, this means 11.01 > 11.001

---------------

As an alternative method, you can subtract the values to see which is larger

The rule or idea is that

(large) - (small) = positive(small) - (large) = negative

For example, 5-2 = 3 showing that 5 is larger than 2. As another example, 7-9 = -2 meaning 7 is smaller than 9. The order of subtraction matters.

In this case, 11.01 - 11.001 = 0.009 showing that 11.01 is larger than 11.001

An angle measuring 60 degrees is measured as 62 degrees. What is the percentage error correct to 3 significant figures?​

Answers

error = 62-60 = 2°

% = (2×100)/60 = 100/30 = 10/3 = 3.333%

In the cafeteria tables are arranged in groups of 4, with each table seating 8 students. How many students can sit at 10 groups of tables?

Answers

4 tables = 1 group. So in 10 groups of tables, there are 40 tables. Each table seats 8 students, so you multiply 8 by 40, and that leaves your answer as 320 students. :)

In a right triangle, Sin (4x + 10) º = Cos (6x) º. What is the value of x?

Answers

Answer:

Step-by-step explanation:

These angles will be equal when each of them measures 45 degrees. On the unit circle, 45 degrees and reference angles of 45 degrees are the only places where sin and cos are the same. This means that

sin(4x + 10) = 45 and cos(6x) = 45

If we add the angles together (the angles being 4x+10 and 6x) we get a 90 degree angle. Thus,

4x + 10 + 6x = 90 and

10x = 80 so

x = 8. Let's try it out:

sin(4(8) + 10) = sin(42) = .6691306064 and

cos(6(8)) = cos(48) = .6691306064

Yay!!!!!

what fraction of 3 hours is 30 minutes?​

Answers

Answer:

7/2

Step-by-step explanation:

In a recent flu epidemic, the number of flu cases increased by 40% each week. There were 3,000 flu cases during the first week of January. How many more people were affected by the flu during the third week of January than during the first week?
1,200
2,880
3,280
4,200

Answers

Answer:

Option B  

(B) 2880   Hope this helps <3

Step-by-step explanation: the total number of people affected by flu during  

the 2nd and 3rd week of January, 1200 + 1680  = 2880

Answer:

1,200

Step-by-step explanation:

40%=.4

(3,000)x(.4)=1,200

1,200 more people were more affected in the third week then the first week

Solve the following problem. It may be helpful to use a chart on scrap paper to organize the information and write the equation. Be sure to show all steps (V.E.S.T.) and work in order to receive full credit.

A grocer wants to make a 10-pound mixture of cashews and peanuts that he can sell for $3.29 per pound. If cashews cost $5.60 per pound and peanuts cost $2.30 per pound, how many pounds of each must he mix?

Answers

Answer:

If cashews cost $5.60 per pound and peanuts cost $2.30 per pound, how many pounds of each must he mix?

Step-by-step explanation:

Weegy: x + y = 1 (we want one pound total) 5.60 * x + 2.30 * y = 3.29 (we want 3.29 per pound) Answer: x = . 3, y = . 7 per one pound To get 10 pounds: cashews - 10x = 3 pounds, peanuts - 10y = 7 pounds.

Type the correct answer in each box. Use numerals instead of words, Simplify the following polynomial expression. (5x2 + 13x – 4) – (17x2 + 7x – 19) + (5x – 7)(3+ + 1)
____ x2 – ____ x + ____​

Answers

Answer:

La primera es la respuesta la segunda parte es la

Step-by-step explanation:

La que te muestra como va el procedimiento

Answer:

-57x^2-10x+8

Step-by-step explanation:

i just did the test on edmentum

Ryan draws a trend line through the following data
points. Did he draw the line correctly?

Answers

Answer: C, You can tell the trend line is incorrect since all of the points are below it. A trend line should be in-between all of the points to give the best estimate.

Step-by-step explanation:

find the product write your question in exponential form 7 ^-1 x 7 ^-7

Answers

Answer:

[tex] {7}^{ - 8} [/tex]

Step-by-step explanation:

[tex] {7}^{ - 1} \times {7}^{ - 7} \\ {7}^{ - 1 + - 7} \\ {7}^{ - 8} [/tex]

Hope this helps you.

x³+y³=(x+y)(x² - xy +y²)​

Answers

Answer:

__________________________________

➡️x3+y3 = (x+y)(x^2-xy+y^2)

--------------------

R.H.S

(x+y)(x^2-xy+y2)

=>x3-x^2y+xy^2+x^2y-y^2x+y3

NOW after cancelling we get :

=> x3+y3

-------------------

L.H.S

=> x3+y3

L.H.S. = R.H.S

Hence VERIFIED✔✔

_______________________________

Step-by-step explanation:

Find an equation in point-slope form for the line having the slope m =1/3 and containing the point (-3,- 6).

Answers

The answer is: y + 6= 1/3 (x + 3)

Help is needed please!!!!

Answers

Answer:

31/20

Step-by-step explanation:

You need to plug in x as y-3.

so instead of 10x+10y=1 it is 10(y-3)+10y=1

Then just simplify the equation to find the value of y

math:John is buying new shoes. He finds the shoes he wants on sale for 30%, and the original price is $88. How much will he pay for the shoes?

Answers

Answer:

26.4

Step-by-step explanation:

hd hd hd hd hd hd hd hd hd hd hd hd

Answer:$61.6

Step-by-step explanation: The sale is 30% so we have to look for 70% of $88 because that's what we're paying for. So, that would be 88 times 70/100 which ends with 61.6.

Other Questions
Translate this phrase into an algebraic expression.RE16 increased by twice a numberUse the variable n to represent the unknown numbers The preferred position doctrine holds that First Amendment freedoms are ____. a applied only to majority opinions b without restriction c more fundamental than other freedoms d subject to interpretation A planet has a circular orbit around a star. It is a distance of 98,000,000 km from the centre of the star. It orbits at an average speed of 45,000 km/h. How many Earth days does it take the planet to orbit the star? Give your answer to 2 sf. Suppose that every day from Monday through Friday,you keep the following routine:You walk directly to school in the moming. On this walk,you take 1500 steps.You play at recess.You play a sport.You walk directly home from school.You think you take about as many steps at recess as youdo playing your sportYourFitBit says that you have taken a total of 38.000 stepsduring the week.How many steps do you take every day at recess? Bengal Co. provides the following sales forecast for the next three months: JulyAugustSeptember Sales units 8,500 9,200 5,950 The company wants to end each month with ending finished goods inventory equal to 20% of the next month's sales. Finished goods inventory on June 30 is 1,700 units. The budgeted production units for August are: identify the value of the fixed cost Can i say that a country has a weak medical field, or does medical field not work?If not what do i replace it with? please solve this fast. plz justify the statement whether it is true or false ayudame y tedoy corona y puntaje Point V is located at -16. Points W and X are each 7 units away from Point V. Where are W and X located? 3 fourths of the over 500000 lakes in Europe are found where? 1)Which of these sentences is punctuated correctly? A) The winners: Ahmed, Hassan, and Mazen are given awards. B) The winners: Ahmed, Hassan, and Mazen, are given awards. 2)Which of these sentences is punctuated correctly? A) "You cannot touch that," her father said, "It's too dangerous." B) "You cannot touch that," her father said, "it's too dangerous." ways in which responsible citizens could bring about awareness on the negative impact of gbv to the victims Why should money be your motivation? Please answer as soon as possible A homeowners association charges a $3.85 fee for each day your payment is late. If a bill was originally $98.75 and paid 13 days late, then how much is owed to the homeowners association? if the probability that it will rain tomorrow is 1/5 what is the probability that it will not rain tomorrow? Zaire became the Democratic Republic of the Congo when Write the electronic configuration of calcium and potassium.